Eine wachsende Funktion f:B→R, B⊂Rf:B→R, B⊂Rf:B\to\mathbb{R},\ B\subset\mathbb{R} muss an jedem Element von BBB außer a stetig sein zählbare Teilmenge von BBB

Ich versuche, die folgende Aussage zu beweisen, aber ich stecke schon eine Weile fest und suche nach einem Hinweis , wie ich es beweisen kann:

"Vermuten B R Und F : B R ist eine steigende Funktion. Beweise das F ist stetig bei jedem Element von B mit Ausnahme einer zählbaren Teilmenge von B ."

Was ich versucht habe:

Da ich die ursprüngliche Aussage nicht angehen konnte, habe ich es versucht, indem ich davon ausgegangen bin B ist auch ein Borel-Set, um die Tatsache in gewisser Weise auszunutzen F muss dann eine von Borel messbare Funktion sein. Dann wenn D := { X B : F  ist bei nicht stetig  X } Und C ist ein Borel-Set, das wir haben F 1 ( C ) ist ein Borel-Set und F 1 ( C ) = ( F 1 ( C ) D ) ( F 1 ( C ) B D ) und an dieser Stelle habe ich versucht, im Widerspruch fortzufahren, das vorausgesetzt D ist unzählbar und versuchte das zu zeigen ( F 1 ( C ) D ) ( F 1 ( C ) B D ) ist kein Borel-Set, aber es ist mir nicht gelungen, daher würde ich mich sehr über Hinweise, Kommentare oder Erklärungen freuen, die mich zu einem fruchtbareren Ansatz anregen könnten, danke .


EDIT (Beweis): Let D := { X B : F  ist bei nicht stetig  X } : seit F monoton (steigend) ist, kann es nur Sprungstellen haben, also wenn D D Dann ICH D := ( lim X D ,   X B F ( X ) , lim X D + ,   X B F ( X ) ) = ( sup X < D ,   X B F ( X ) , inf X > D ,   X B F ( X ) ) ist ein nicht leeres Intervall in R also muss es eine rationale Zahl geben Q D darin, was wir wählen können und da ICH D ICH D ' = für D D ' * Diese rationale Zahl ist ebenfalls eindeutig, sodass wir eine injektive Funktion definieren können G : D Q , G ( D ) := Q D . Daher # ( D ) # ( Q ) = # ( N ) dh die Menge der Punkte, an denen F ist diskontinuierlich ist (höchstens) zählbar, wie gewünscht.


(*) Vermuten ICH D ICH D ' für einige D D ' (wir können davon ausgehen, dass D < D ' ): dann muss es sein j ICH D ICH D ' So sup X < D ,   X B F ( X ) < j < inf X > D ,   X B F ( X ) Und sup X < D ' ,   X B F ( X ) < j < inf X > D ' ,   X B F ( X ) was impliziert (seit D D ' inf X > D ,   X B F ( X ) sup X < D ' , X B F ( X ) ) Das j < j , Widerspruch.

Antworten (2)

Wenn F ist bei nicht stetig X B , Dann inf { F ( T ) T B , T > X } > sup { F ( T ) T B , T < X } (einschließlich der Möglichkeit inf = + oder sup = ). Wählen Sie eine rationale dazwischen inf Und sup und erhalten so eine injektive Abbildung aus der Menge der Diskontinuitäten to Q .

Lasst uns annehmen B = ( A , B ) . Wenn X < j < z Dann F ( X ) F ( j ) F ( z ) und jede Diskontinuität muss eine Sprungdiskontinuität sein. Wenn J N ist die Menge der Sprünge mit der Größe > 1 N , Dann J N ist endlich und der Beweis folgt. QED